Skip to main content
\(\require{mathrsfs}\newcommand{\abs}[1]{\left| #1 \right|} \renewcommand{\emptyset}{\varnothing} \DeclareMathOperator{\dom}{dom} \DeclareMathOperator{\range}{rng} \DeclareMathOperator{\perm}{perm} \newcommand{\lt}{<} \newcommand{\gt}{>} \newcommand{\amp}{&} \)

Section5.8Exam 2 Review

1

Prove that \(\{ 5m+2 \mid m \in \mathbb{Z} \} = \{ 5n-3 \mid n \in \mathbb{Z} \}\text{.}\)

Solution

Call the left-hand set \(A\) and the right-hand set \(B\text{.}\)

(\(\subseteq\)). Suppose that \(x \in A\text{.}\) Then \(x = 5m+2\) for some \(m \in \mathbb{Z}\text{.}\) Thus \(x = 5m+2+3-3 = 5(m+1)-3\text{.}\) Since \(m+1 \in \mathbb{Z}\text{,}\) we find \(x \in B\text{.}\)

(\(\supseteq\)). Suppose that \(x \in B\text{.}\) Then \(x = 5n-3\) for some \(n \in \mathbb{Z}\text{.}\) Thus \(x = 5n-3-2+2 = 5(n-1)+2\text{.}\) Since \(n-1 \in \mathbb{Z}\text{,}\) we find \(x \in A\text{.}\)

2

For an integer \(m \in \mathbb{Z}\text{,}\) define \(m\mathbb{Z} := \{ x \mid x = mk, \text{for some}\ k\in\mathbb{Z} \}\text{.}\) In English, \(m\mathbb{Z}\) is the set of multiples of \(m\text{.}\) Prove that \(m\mathbb{Z} \subseteq n\mathbb{Z}\) if and only if \(n \mid m\text{.}\)

Solution

Let \(m,n \in \mathbb{Z}\) be arbitrary.

(\(\Rightarrow\)). Suppose that \(m\mathbb{Z} \subseteq n\mathbb{Z}\text{.}\) Since \(m = m\cdot 1 \in m\mathbb{Z}\text{,}\) we know that \(m \in n\mathbb{Z}\text{.}\) Therefore \(m = nk\) for some \(k \in \mathbb{Z}\text{.}\) Thus \(n \mid m\text{.}\)

(\(\Leftarrow\)). Suppose that \(n \mid m\text{.}\) Then \(m = nk\) for some \(k \in \mathbb{Z}\text{.}\) Now take any element \(x \in m\mathbb{Z}\text{.}\) By definition of \(m\mathbb{Z}\text{,}\) we find \(x = mj\) for some integer \(j\text{.}\) Hence \(x = mj = (nk)j = n(kj)\text{.}\) Since \(kj\) is an integer, we find \(x \in n\mathbb{Z}\text{.}\)

3

On pages 86–87 of Section 2.2, do problems 6, 9, 10, 15(a), (c), (d), 16(a).

Solution

#6.

  1. \(A = \{1,2\}, B = \{2\}, C = \{1\}\)
  2. \(A = B = C = \{1\}\)
  3. \(A = B = \{1\}, C = \{1,2\}\)
  4. \(A = \{1,3\}, B = \{2,3\}, C = \{3\}\)
  5. \(A = \{1\}, B = C = \{1,2\}\)
  6. \(A = B = C = \{1\}\)

#9.

  1. Note that \(A \subseteq B\) if and only if \(\forall x, x \in A \implies x \in B\) if and only if \(\forall x, \neg( x \in A \wedge x \notin B)\) if and only if \(\forall x, \neg( x\in A \setminus B)\) if and only if \(\forall x, x \notin A \setminus B\) if and only if \(A \setminus B = \varnothing\text{.}\) Note that the second “if and only if” uses the equivalence \((P \implies Q) \iff \neg(P \wedge \neg Q)\text{,}\) and the last “if and only if” uses the definition of the empty set.
  2. Suppose \(A \subseteq B \cup C\) and \(A \cap B = \varnothing\text{.}\) Take an \(x \in A\text{.}\) Then since \(A \subseteq B \cup C\text{,}\) \(x \in B \cup C\text{,}\) so either \(x \in B\) or \(x \in C\text{.}\) But since \(A \cap B = \varnothing\text{,}\) it cannot be the case that \(x \in B\) (for otherwise \(x \in A \cap B\)), and therefore \(x \in C\text{.}\) Hence \(A \subseteq C\text{.}\)
  3. (\(\Rightarrow\)) Suppose that \(C \subseteq A \cap B\text{.}\) Since \(A \cap B \subseteq A\) and \(A \cap B \subseteq B\text{,}\) we can use the small lemma we proved in class to conclude \(C \subseteq A\) and \(C \subseteq B\text{.}\) (\(\Leftarrow\)) Suppose that \(C \subseteq A\) and \(C \subseteq B\text{.}\) Now take any \(x \in C\text{.}\) By the first condition, \(x \in A\text{,}\) and by the second condition, \(x \in B\text{.}\) Therefore \(x \in A \cap B\text{.}\) Thus \(C \subseteq A \cap B\text{.}\)
  4. Suppose \(A \subseteq B\text{.}\) Now take any \(x \in A \setminus C\text{.}\) Then \(x \in A\) and \(x \notin C\text{.}\) Since \(A \subseteq B\text{,}\) we have \(x \in B\text{.}\) Thus \(x \in B \setminus C\text{.}\) Therefore \(A \setminus C \subseteq B \setminus C\text{.}\)
  5. Notice that for any \(x\text{,}\) \begin{align*} x \in (A \setminus B) \setminus C &\iff x \in A \setminus B \wedge x \notin C\\ &\iff x \in A \wedge x \notin B \wedge x \notin C\\ &\iff (x \in A \wedge x \notin C \wedge x \notin B) \vee (x \in A \wedge x \notin C \wedge x \in C)\\ &\iff (x \in A \wedge x \notin C) \wedge (x \notin B \vee x \in C)\\ &\iff (x \in A \wedge x \notin C) \wedge \neg(x \in B \wedge x \notin C)\\ &\iff x \in (A \setminus C) \wedge x \notin (B \setminus C)\\ &\iff x \in (A \setminus C) \setminus (B \setminus C) \end{align*}
  6. Suppose \(A \subseteq C\) and \(B \subseteq C\text{.}\) Take any \(x \in A \cup B\text{.}\) Then either \(x \in A\) or \(x \in B\text{.}\) If \(x \in A\text{,}\) then \(x \in C\) since \(A \subseteq C\text{.}\) If \(x \in B\text{,}\) then \(x \in C\) since \(B \subseteq C\text{.}\) Either way, \(x \in C\text{.}\) Thus \(A \cup B \subseteq C\text{.}\)
  7. Take any \(x \in (A \cup B) \cap C\text{.}\) Then \(x \in A \cup B\) and \(x \in C\text{.}\) So, either \(x \in A\) or \(x \in B\text{.}\) Case 1: \(x \in A\). Then certainly \(x \in A \cup (B \cap C)\text{.}\) Case 2: \(x \in B\). Then since \(x \in C\) also, we have \(x \in B \cap C\text{.}\) Hence \(x \in A \cup (B \cap C)\text{.}\)
  8. To show \((A \setminus B) \cap B = \varnothing\text{,}\) we will use a proof by contradiction. That is, suppose this set is nonempty. Then there is some \(x \in (A \setminus B) \cap B\text{,}\) so \(x \in A \setminus B\) and \(x \in B\text{.}\) But the first statement means \(x \in A\) and \(x \notin B\text{,}\) contradicting the fact that \(x \in B\text{.}\) Hence our assumption (that \((A \setminus B) \cap B \not= \varnothing\)), was false. Therefore \((A \setminus B) \cap B = \varnothing\text{.}\)

#10. Let \(A,B,C,D\) be sets.

  1. Suppose \(C \subseteq A\) and \(D \subseteq B\text{.}\) Take any \(x \in C \cap D\text{.}\) Then \(x \in C\) and \(x \in D\text{.}\) Since \(C \subseteq A\) we have \(x \in A\text{.}\) Since \(D \subseteq B\) we have \(x \in B\text{.}\) Thus \(x \in A \cap B\text{.}\)
  2. Suppose \(C \subseteq A\) and \(D \subseteq B\text{.}\) Take any \(x \in C \cup D\text{.}\) Then \(x \in C\) or \(x \in D\text{.}\) Case 1: suppose \(x \in C\text{.}\) Since \(C \subseteq A\) we have \(x \in A\text{.}\) Case 2: suppose \(x \in D\text{.}\) Since \(D \subseteq B\) we have \(x \in B\text{.}\) So \(x \in A\) or \(x \in B\text{.}\) Thus \(x \in A \cup B\text{.}\)
  3. Suppose \(C \subseteq A\) and \(D \subseteq B\text{,}\) and \(A,B\) are disjoint. Then \(A \cap B = \varnothing\text{,}\) and by #10(a), \(C \cap D \subseteq A \cap B = \varnothing\text{.}\) But trivially \(\varnothing \subseteq C \cap D\text{,}\) so they must be equal.
  4. Suppose \(C \subseteq A\) and \(D \subseteq B\text{.}\) Take any \(x \in D \setminus A\text{.}\) Then \(x \in D\) and \(x \notin A\text{.}\) Since \(D \subseteq B\) we have \(x \in B\text{.}\) Since \(C \subseteq A\text{,}\) by an earlier problem, we have \(x \notin C\text{.}\) Thus \(x \in B \setminus C\text{.}\)
  5. Suppose \(A \cup B \subseteq C \cup D\text{,}\) \(A \cap B = \varnothing\text{,}\) and \(C \subseteq A\text{.}\) Take \(x \in B\text{,}\) then \(x \in A \cup B\text{,}\) and hence \(x \in C \cup D\text{.}\) Since \(A \cap B = \varnothing\text{,}\) \(x \notin A\text{,}\) and hence \(x \notin C\text{.}\) But because \(x \in C \cup D\) and \(x \notin C\text{,}\) we must have \(x \in D\text{.}\) Therefore \(B \subseteq D\text{.}\)
4

For each natural number \(n\text{,}\) let \(A_n = \{ 5n,5n+1,\ldots,6n\}\text{.}\) Find the union and intersection over \(\mathcal{A} := \{ A_n \mid n \in \mathbb{N} \}\text{.}\) Be sure to prove your claim.

Solution

Claim: \(\bigcup_{n=1}^{\infty} A_n = B \cup C\) where \(B := \{5,6,10,11,12,15,16,17,18\}\) and \(C := \{ n \in \mathbb{N} \mid n \ge 20\}\text{.}\)

“\(\subseteq\)” Suppose \(x \in \bigcup_{n=1}^{\infty} A_n\text{.}\) Then there is some \(k \in \mathbb{N}\) so that \(x \in A_k\text{.}\)

Case 1: \(k=1,2,3\text{.}\) Then \(x \in A_k \subseteq A_1 \cup A_2 \cup A_3 = B\text{,}\) and hence \(x \in B \cup C\text{.}\)

Case 2: \(k \ge 4\text{.}\) Then \(20 = 5 \cdot 4 \le 5k \le x \le 6k\text{,}\) and hence \(x \in C\text{,}\) so \(x \in B \cup C\text{.}\)

“\(\supseteq\)” Suppose that \(x \in B \cup C\text{.}\) If \(x \in B = A_1 \cup A_2 \cup A_3\text{,}\) then clearly \(x \in \bigcup_{n=1}^{\infty} A_k\text{.}\)

If \(x \in C\text{,}\) then by the division algorithm we can write \(x = 5m + r\) with \(m,r \in \mathbb{Z}\) and \(0 \le r \lt 5\text{.}\) Moreover, since \(20 = 5 \cdot 4\) and \(x \ge 20\) we know \(m \ge 4\text{.}\) Furthermore, \(x = 5m+r \le 5m + 4 \le 5m + m = 6m\text{.}\) Thus \(5m \le x \le 6m\text{.}\) And therefore \(x \in A_m\text{,}\) so \(x \in \bigcup_{n=1}^{\infty} A_n\text{.}\)

5

Give an example of an indexed collection of sets \(\{ A_n \mid n \in \mathbb{N} \}\) which are pairwise disjoint and for which \(A_n \subseteq (0,1)\) (the open interval). Be sure to prove your collection satisfies the listed properties.

Solution

Simple example: set \(A_n = \{\frac{1}{n+1}\}\text{.}\) Clearly these are pairwise disjoint because each only contains one element and none of those are the same one. Moreover, \(0 \lt \frac{1}{n+1} \lt 1\text{,}\) so \(A_n \subseteq (0,1)\text{.}\)

6

Give an example of an indexed collection of sets \(\{ A_n \mid n \in \mathbb{N} \}\) no two of which are disjoint and such that each \(A_n \subseteq (0,1)\) (the open interval), but \(\displaystyle\bigcap_{n \in \mathbb{N}} A_n = \emptyset\text{.}\) Be sure to prove your collection satisfies the listed properties.

Solution

Consider \(A_n = (0,\frac{1}{n})\text{.}\) Clearly, if \(n \lt m\text{,}\) then \(\frac{1}{m} \lt \frac{1}{n}\text{,}\) so \(A_m \subseteq A_n\) and thus \(A_m \cap A_n = A_m \not= \emptyset\text{.}\)

We now prove the intersection over all the sets is empty. Let \(x \in \mathbb{R}\text{.}\) If \(x \le 0\text{,}\) then \(x \notin A_1\text{,}\) so \(x\) is not in the intersection either. Now suppose \(x \gt 0\text{.}\) Then by the Archimedean Principle, there is some \(n \in \mathbb{N}\) so that \(\frac{1}{n} \lt x\text{.}\) Therefore \(x \notin A_n\) and hence \(x\) is not in the intersection either. This proves that every \(x\) is not in the intersection, so the intersection is empty.

7

Let \(\{ A_\alpha \mid \alpha \in I \}\) be an indexed family of sets and let \(B\) be any set. Prove that

  1. \(B \cap \bigcup_{\alpha \in I} A_{\alpha} = \bigcup_{\alpha \in I} (B \cap A_{\alpha})\)
  2. \(B \cup \bigcap_{\alpha \in I} A_{\alpha} = \bigcap_{\alpha \in I} (B \cup A_{\alpha})\)
Solution1

Subset based solution.

“\(\subseteq\)” Suppose that \(x \in B \cap \bigcup_{\alpha \in I} A_{\alpha}\text{,}\) then \(x \in B\) and \(x \in \bigcup_{\alpha \in I} A_{\alpha}\text{.}\) Thus there is some \(\beta \in I\) so that \(x \in A_{\beta}\text{.}\) Therefore \(x \in B \cap A_{\beta}\text{.}\) Hence \(x \in \bigcup_{\alpha \in I} (B \cap A_{\alpha})\text{.}\)

“\(\supseteq\)” Suppose that \(x \in \bigcup_{\alpha \in I} (B \cap A_{\alpha})\text{,}\) then there is some \(\beta \in I\) so that \(x \in B \cap A_{\beta}\text{.}\) Thus \(x \in B\) and \(x \in B \cap A_{\beta}\text{.}\) Hence \(x \in \bigcup_{\alpha \in I} A_{\alpha}\text{.}\) Therefore \(x \in B \cap \bigcup_{\alpha \in I} A_{\alpha}\text{.}\)

The equality \(B \cup \bigcap_{\alpha \in I} A_{\alpha} = \bigcap_{\alpha \in I} (B \cup A_{\alpha})\) follows from a similar proof.

Solution2

Iff based solution.

Note that

\begin{align*} x \in B \cap \bigcup_{\alpha \in I} A_{\alpha} &\iff (x \in B) \wedge (\exists \alpha \in I)(x \in A_{\alpha})\\ &\iff (\exists \alpha \in I)\big((x \in B) \wedge (x \in A_{\alpha})\big)\\ &\iff (\exists \alpha \in I)(x \in B \cap A_{\alpha})\\ &\iff x \in \bigcup_{\alpha \in I} (B \cap A_{\alpha}), \end{align*}

and

\begin{align*} x \in B \cup \bigcap_{\alpha \in I} A_{\alpha} &\iff (x \in B) \vee (\forall \alpha \in I)(x \in A_{\alpha})\\ &\iff (\forall \alpha \in I)\big((x \in B) \vee (x \in A_{\alpha})\big)\\ &\iff (\forall \alpha \in I)(x \in B \cup A_{\alpha})\\ &\iff x \in \bigcap_{\alpha \in I} (B \cup A_{\alpha}). \end{align*}
8

For each natural number \(n\text{,}\) define \(A_n := \{ n,n+1,n+2,\ldots,\}\) (i.e., \(A_n = \{ k \in \mathbb{N} \mid k \ge n \}\)). Determine \(\bigcap_{n=1}^\infty A_n\) and prove your claim.

Solution

We claim that \(\bigcap_{n=1}^\infty A_n = \emptyset\text{.}\)

Since our universe is \(\mathbb{N}\text{,}\) it suffices to show that every \(k \in \mathbb{N}\) is not in this intersection. So, let \(k \in \mathbb{N}\) be arbitrary. Notice that \(k \lt k+1\text{,}\) so \(k \not\ge k+1\text{.}\) Therefore \(k \notin A_{k+1}\text{.}\) Hence \(k \notin \bigcap_{n=1}^{\infty} A_n\text{.}\)

9

(Pg. 110, #5(c,d,e,f)). Give an inductive definition for each:

  • \(\{n \mid n = 2^k\ \text{for some}\ k \in \mathbb{N} \}\)
  • \(\{a,a+d,a+2d,a+3d,\ldots\}\text{,}\) where \(a,d \in \mathbb{R}\text{.}\) (The elements in this set form an arithmetic progression.
  • \(\{a,ar,ar^2,ar^3,\ldots\}\text{,}\) where \(a,d \in \mathbb{R}\text{.}\) (The elements in this set form an geometric progression.
  • \(\bigcup_{i=1}^n A_i\text{,}\) for some indexed family \(\{ A_i \mid i \in \mathbb{N} \}\text{.}\)
Solution

  • \(\{n \mid n = 2^k\ \text{for some}\ k \in \mathbb{N} \} := \{a_n \mid n \in \mathbb{N}\}\text{,}\) where \(a_1 := 2\) and \(a_{n+1} = 2 a_n\text{.}\)
  • \(\{a,a+d,a+2d,a+3d,\ldots\} := \{a_n \mid n \in \mathbb{N}\}\text{,}\) where \(a_1 := a\) and \(a_{n+1} = a_n + d\text{.}\)
  • \(\{a,ar,ar^2,ar^3,\ldots\} := \{a_n \mid n \in \mathbb{N}\}\text{,}\) where \(a_1 := a\) and \(a_{n+1} = a_n r\text{.}\)
  • \(\bigcup_{i=1}^n A_i\text{,}\) for some indexed family \(\{ A_i \mid i \in \mathbb{N} \}\text{.}\) \(\bigcup_{i=1}^1 A_i := A_1\text{,}\) and \(\bigcup_{i=1}^{n+1} A_i := \left( \bigcup_{i=1}^n A_i \right) \cup A_{n+1}\text{.}\)
10

(Pg. 110, #6(d)). Use induction to prove that for all \(n \in \mathbb{N}\text{,}\)

\begin{equation*} \sum_{j=1}^n j \cdot j! = (n+1)! - 1. \end{equation*}
Solution

For the base case, notice that

\begin{equation*} \sum_{j=1}^1 j \cdot j! = 1 \cdot 1! = 1 = 2-1 = (1+1)! - 1. \end{equation*}

Now let \(k \in \mathbb{N}\) be arbitrary and assume \(\sum_{j=1}^k j \cdot j! = (k+1)! - 1\text{.}\) Then we find

\begin{align*} \sum_{j=1}^{k+1} j \cdot j! &= \sum_{j=1}^k j \cdot j! + (k+1) \cdot (k+1)!\\ &= ((k+1)! - 1) + (k+1) \cdot (k+1)!\\ &= (1+k+1)(k+1)! - 1\\ &= (k+2)! - 1 \end{align*}
11

(Pg. 110, #6(g)). Use induction to prove that for all \(n \in \mathbb{N}\text{,}\)

\begin{equation*} \sum_{j=1}^n \frac{1}{j(j+1)} = \frac{n}{n+1}. \end{equation*}
Solution

For the base case, notice that

\begin{equation*} \sum_{j=1}^1 \frac{1}{j(j+1)} = \frac{1}{1\cdot 2} = \frac{1}{2} = \frac{1}{1+1}. \end{equation*}

Now let \(k \in \mathbb{N}\) be arbitrary and assume \(\sum_{j=1}^k \frac{1}{j(j+1)} = \frac{k}{k+1}\text{.}\) Then we find

\begin{align*} \sum_{j=1}^{k+1} \frac{1}{j(j+1)} &= \sum_{j=1}^k \frac{1}{j(j+1)} + \frac{1}{k(k+1)}\\ & \frac{k}{k+1} + \frac{1}{k(k+1)}\\ & \frac{k(k+2)}{(k+1)(k+2)} + \frac{1}{(k+1)(k+2)}\\ & \frac{k^2 + 2k + 1}{(k+1)(k+2)}\\ & \frac{(k+1)^2}{(k+1)(k+2)}\\ & \frac{(k+1)}{(k+2)} \end{align*}
12

(Pg. 110, #6(j)). Use induction to prove that for all \(n \in \mathbb{N}\text{,}\)

\begin{equation*} \prod_{j=1}^n \left( 1 - \frac{1}{j+1} \right) = \frac{1}{n+1}. \end{equation*}
Solution

For the base case, notice that

\begin{equation*} \prod_{j=1}^1 \left( 1 - \frac{1}{j+1} \right) = 1 - \frac{1}{1+1} = \frac{1}{1+1}. \end{equation*}

Now let \(k \in \mathbb{N}\) be arbitrary and assume \(\prod_{j=1}^k \left( 1 - \frac{1}{j+1} \right) = \frac{1}{n+1}\text{.}\) Then we find

\begin{align*} \prod_{j=1}^{k+1} \left( 1 - \frac{1}{j+1} \right) &= \left( \prod_{j=1}^k \left( 1 - \frac{1}{j+1} \right) \right) \cdot \left( 1 - \frac{1}{k+2} \right)\\ & \frac{1}{k+1} \cdot \left( \frac{k+2}{k+2} - \frac{1}{k+2} \right)\\ & \frac{1}{k+1} \cdot \frac{k+1}{k+2}\\ & \frac{1}{k+2}. \end{align*}
13

(Pg. 111, #6(k)). Use induction to prove that for all \(n \in \mathbb{N}\text{,}\)

\begin{equation*} \prod_{j=1}^n (2i-1) = \frac{(2n)!}{n!2^n}. \end{equation*}
Solution

For the base case, notice that

\begin{equation*} \prod_{j=1}^1 (2j-1) = 2\cdot 1 - 1 = 1 = \frac{2 \cdot 1}{1 \cdot 2} = \frac{2!}{1! 2^1}. \end{equation*}

Now let \(k \in \mathbb{N}\) be arbitrary and assume \(\prod_{j=1}^k (2j-1) = \frac{(2k)!}{k!2^k}\text{.}\) Then we find

\begin{align*} \prod_{j=1}^{k+1} (2j-1) &= \left( \prod_{j=1}^k (2j-1) \right) \cdot \big( (2(k+1)-1\big)\\ & \frac{(2k)!}{k!2^k} \cdot (2k+1)\\ & \frac{(2k+1)!}{k!2^k} \cdot \frac{2k+2}{(k+1)2}\\ & \frac{(2k+2)!}{(k+1)!2^{k+1}}\\ & \frac{(2(k+1))!}{(k+1)!2^{k+1}}. \end{align*}
14

(Pg. 111, #6(l)). (Sum of finite geometric series.) Use induction to prove that for all \(n \in \mathbb{N}\text{,}\)

\begin{equation*} \sum_{j=0}^{n-1} ar^j = \frac{a(r^n - 1)}{r-1}. \end{equation*}
Solution

For the base case, notice that

\begin{equation*} \sum_{j=0}^{1-1} ar^j = ar^0 = a = \frac{a(r^1 - 1)}{r-1}. \end{equation*}

Now let \(k \in \mathbb{N}\) be arbitrary and assume \(\sum_{j=0}^{k-1} ar^j = \frac{a(r^k-1)}{r-1}\text{.}\) Then we find

\begin{align*} \sum_{j=0}^k ar^j &= \sum_{j=0}^{k-1} ar^j + ar^k\\ &= \frac{a(r^k-1)}{r-1} + ar^k\\ &= \frac{ar^k-a}{r-1} + \frac{ar^k(r-1)}{r-1}\\ &= \frac{ar^k-a}{r-1} + \frac{ar^{k+1}-ar^k}{r-1}\\ &= \frac{ar^{k+1}-a}{r-1}\\ &= \frac{a(r^{k+1}-1)}{r-1} \end{align*}
15

(Pg. 111, #7(g)). Use induction to prove that for all \(n \in \mathbb{N}\text{,}\) \(8\) divides \(9^n - 1\)

Solution

For the base case, notice that \(8 = 1 \cdot 8\text{,}\) so \(8 \mid 8 = 9^1 -1\text{.}\) Now let \(k \in \mathbb{N}\) be arbitrary and assume \(8 \mid 9^k-1\text{.}\) Then there is some \(j \in \mathbb{Z}\) for which \(9^k-1 = 8j\text{.}\) Thus

\begin{equation*} 9^{k+1}-1 = 9\cdot 9^k - 1 = 9(8j+1) - 1 = 8(9j)+9-1 = 8(9j+1). \end{equation*}

Since \(9j+1 \in \mathbb{Z}\text{,}\) we have shown \(8 \mid 9^{k+1}-1\text{.}\) By induction we have proven the result.

16

(Pg. 111, #7(k)). Use induction to prove that for all \(n \in \mathbb{N}\text{,}\)

\begin{equation*} \sum_{j=1}^n \frac{1}{j^2} \le 2 - \frac{1}{n}. \end{equation*}
Solution

For the base case, notice that

\begin{equation*} \sum_{j=1}^1 \frac{1}{j^2} = \frac{1}{1^2} = 1 = 2-\frac{1}{1}. \end{equation*}

Now let \(k \in \mathbb{N}\) be arbitrary and assume \(\sum_{j=1}^k \frac{1}{j^2} \le 2-\frac{1}{k}\text{.}\) Before we continue, recall the following elementary fact: for any \(m \in \mathbb{N}\text{,}\) \(\frac{1}{m(m+1)} = \frac{1}{m}-\frac{1}{m+1}\text{.}\) Then we find

\begin{align*} \sum_{j=1}^{k+1} \frac{1}{j^2} &= \sum_{j=1}^k \frac{1}{j^2} + \frac{1}{(k+1)^2}\\ &\le \left( 2 - \frac{1}{k} \right) + \frac{1}{(k+1)^2}\\ &\le \left( 2 - \frac{1}{k} \right) + \frac{1}{k(k+1)}\\ &\le \left( 2 - \frac{1}{k} \right) + \left( \frac{1}{k} - \frac{1}{k+1} \right)\\ &= 2 - \frac{1}{k+1} \end{align*}

By induction we have proven the result.

17

(Pg. 111, #7(l)). Use induction to prove that for all \(n \in \mathbb{N}\text{,}\) and every positive real number \(x\text{,}\)

\begin{equation*} (1+x)^n \ge 1+nx. \end{equation*}
Solution

For the base case, notice that \((1+x)^1 = 1+x = 1+1x\text{.}\) Now let \(k \in \mathbb{N}\) be arbitrary and assume \((1+x)^k \ge 1+kx\text{.}\) Then we find (be careful! the first inequality only works because \(1+x>0\) since \(x>0\text{.}\) When you start introducing other variables, you have to pay attention to such things!)

\begin{align*} (1+x)^{k+1} &= (1+x)^k (1+x)\\ &\ge (1+kx)(1+x)\\ &= 1+kx+x+kx^2\\ &= 1+(k+1)x + kx^2\\ &\gt 1+(k+1)x, \end{align*}

where the last inequality follows because \(k \in \mathbb{N}\) and so \(kx^2>0\text{.}\) By induction we have proven the result.

18

(Pg. 111, #7(n)). Assume the differentiation formulas \(\frac{d}{dx}(x) = 1\) and \(\frac{d}{dx}(fg) = f \frac{dg}{dx} + g \frac{df}{dx}\) (product rule). Use induction to prove that for all \(n \in \mathbb{N}\text{,}\) \(\frac{d}{dx} (x^n) = nx^{n-1}\text{.}\)

Solution

The base case is one of our assumptions, so we are done. Now let \(k \in \mathbb{N}\) be arbitrary and assume \(\frac{d}{dx}(x^k) = kx^{k-1}\text{.}\) Then notice that \(x^{k+1} = x^k \cdot x\text{,}\) so applying the product rule to the functions \(f(x) = x^k\) and \(g(x) = x\text{,}\) we find

\begin{align*} \frac{d}{dx} (x^{k+1}) &= \frac{d}{dx} (x^k \cdot x)\\ &= x^k \frac{d}{dx}(x) + x \frac{d}{dx} (x^k)\\ &= x^k \cdot 1 + x \cdot kx^{k-1}\\ &= x^k + kx^k\\ &= (k+1)x^k. \end{align*}

By induction we have proven the result.

19

(Pg. 111, #8(g)). Use generalized induction to prove that for all \(n \gt 2\text{,}\) the sum of the angle measures of the interior angles of a convex polygon of \(n\) sides is \((n-2) \pi\) (measured in radians).

Hint

Use a standard fact from geometry.

Solution

The base case, when \(n=3\) is just the standard geometry fact that the sum of the interior angle measures of a triangle is always \(\pi\text{.}\)

Now suppose \(k \in \mathbb{N}\) and \(k \gt 2\) (i.e., \(k \ge 3\)) and the sum of the interior angle measures of any \(k\)-sided polygon is \((k-2) \pi\text{.}\) Then consider any \((k+1)\)-sided polygon. Pick a vertex \(v\) with an interior angle less than \(\pi\text{.}\) Since \(k+1 \ge 4\text{,}\) we can divide our polygon into a triangle and a \(k\)-sided polygon by drawing a line between the vertices adjacent to \(v\text{.}\) Moreover, the sum of the interior angles of the original \((k+1)\)-gon is the same as the sum of the interior angles of the \(k\)-gon (which has total angle measure \((k-2) \pi\)) added to the sum of the interior angles of the triangle (which has total angle measure \(\pi\text{.}\) Therefore, the total angle measure for the \((k+1)\)-gon is \((k-2) \pi + \pi = ((k+1)-2) \pi\text{,}\) as desired.

20

(Pg. 153–154, #1(f,h,k,m)).

Solution

For each relation, determine if it is reflexive, symmetric or transitive.

  1. #1(f). The relation is \(\not=\) on \(\mathbb{N}\text{.}\) This is not reflexive since \(x = x\) for any \(x \in \mathbb{N}\text{.}\) This relation is symmetric since \(x \not= y\) if and only if \(y \not= x\text{.}\) This relation is not transitive on \(\mathbb{N}\) since \(0 \not= 1\) and \(1 \not= 0\text{,}\) but \(0 = 0\text{.}\)
  2. #1(h). The relation is \(R := \{ (x,y) \in \mathbb{Z}^2 \mid x+y = 10 \}\) on \(\mathbb{Z}\text{.}\) This relation is not reflexive since \(0+0 = 0 \not= 10\) and hence \((0,0) \notin R\text{.}\) This relation is symmetric since \((x,y) \in R\) if and only if \(x+y = 10\) if and only if \(y+x = 10\) if and only if \((y,x) \in R\text{.}\) This relation is not transitive since \((4,6) \in R\) and \((6,4) \in R\text{,}\) but \((4,4) \notin R\text{.}\)
  3. #1(k). The relation is \(R\) on \(\mathbb{R}^2\) where \((x,y)\ R\ (z,w)\) if and only if \(x+z \le y+w\text{.}\) This relation is not reflexive since \(1+1 \not\le 0+0\text{,}\) so \((1,0)\ \not R\ (1,0)\text{.}\) This relation is symmetric since \((x,y)\ R\ (z,w)\) if and only if \(x+z \le y+w\) if and only if \(z+x \le w+y\) if and only if \((z,w)\ R\ (x,y)\text{.}\) This relation is not transitive since \((1,0)\ R\ (0,1)\) and \((0,1)\ R\ (1,0)\text{,}\) but \((1,0)\ \not R\ (1,0)\text{.}\)
  4. #1(m). The relation is \(T\) on \(\mathbb{R}^2\) where \((x,y)\ R\ (z,w)\) if and only if \(x+y \le z+w\text{.}\) This relation is reflexive since for any \((x,y) \in \mathbb{R}^2\text{,}\) \(x+y \le x+y\) and so \((x,y)\ T\ (x,y)\text{.}\) This relation is not symmetric since \(0+0 \le 1+1\) but \(1+1 \not\le 0+0\text{,}\) so \((0,0)\ T\ (1,1)\) but \((1,1)\ \not T\ (0,0)\text{.}\) This relation is transitive. Indeed, if \((x,y)\ T\ (z,w)\) and \((z,w)\ T\ (a,b)\text{,}\) then \(x+y \le z+w\) and \(z+w \le a+b\text{,}\) and therefore \(x+y \le a+b\text{.}\) Thus \((x,y)\ T\ (a,b)\) and so \(T\) is transitive.
21

(Pg. 154, #5(a)).

Solution

We will prove that the relation \(R\) on \(\mathbb{R}\) given by \(x\ R\ y\) if and only if \(x - y \in \mathbb{Q}\) is an equivalence relation. Note that this relation is reflexive since for any \(x \in \mathbb{R}\text{,}\) \(x - x = 0 \in \mathbb{Q}\text{,}\) and hence \(x\ R\ x\text{.}\) This relation is also symmetric since for any \(x,y \in \mathbb{R}\text{,}\) we have \(x-y \in \mathbb{Q}\) if and only if \(y-x = -(x-y) \in \mathbb{Q}\text{.}\) This relation is also transitive since for any \(x,y,z \in \mathbb{R}\text{,}\) if \(x\ R\ y\) and \(y\ R\ z\text{,}\) then \(x-y, y-z \in \mathbb{Q}\text{,}\) and hence \(x-z = (x-y)+(y-z) \in \mathbb{Q}\text{.}\)

We give the equivalence classes of \(0,1,\sqrt{2}\) below.

\begin{gather*} [0] = [1] = \mathbb{Q}\\ [\sqrt{2}] = \{ x+\sqrt{2} \mid x \in \mathbb{Q} \} \end{gather*}
22

(Pg. 154, #5(c)).

Solution

We will prove that the relation \(V\) on \(\mathbb{R}\) given by \(x\ V\ y\) if and only if \(x = y\) or \(xy = 1\) is an equivalence relation.

This relation is reflexive since for any \(x \in \mathbb{R}\text{,}\) \(x = x\) and hence \(x\ V\ x\text{.}\) This relation is symmetric since \(x = y\) or \(xy = 1\) if and only if \(y = x\) or \(yx = 1\text{.}\) This relation is also transitive, for if \(x,y,z \in \mathbb{R}\) and \(x\ V\ y\) and \(y\ V\ z\text{,}\) then if either \(x = y\) or \(y = z\text{,}\) the conclusion \(x\ V\ z\) is trivial. So suppose \(x \not= y \not= z\text{.}\) Thus \(xy = 1 = yz\text{,}\) and so none of \(x,y,z\) are zero. Dividing by \(z\) we obtain \(x = z\text{.}\) Thus \(x\ V\ z\text{.}\)

We give the equivalence classes of \(3,-\frac{2}{3},0\) below.

\begin{equation*} [3] = \left\{ 3,\frac{1}{3} \right\} \quad \left[ -\frac{2}{3} \right] = \left\{ -\frac{2}{3}, -\frac{3}{2} \right\} \quad [0] = \{0\}. \end{equation*}
23

(Pg. 155, #5(h)).

Solution

Consider the relation \(R\) on the set of differentiable functions defined by \(f\ R\ g\) if and only if \(f\) and \(g\) have the same first derivative. This relation is obviously an equivalence relation because it is a notion of “sameness.” By the Fundamental Theorem of Calculus, the elements of any equivalence class are things of the form \(f + C\) where \(C\) is an arbitrary constant.

Fibonacci sequence

Consider the following inductively (recursively) defined sequence. Let \(f_1 = 1, f_2 = 1\) and for all \(n \ge 3\text{,}\) let \(f_n = f_{n-1} + f_{n-2}\text{.}\) The sequence of positive integers \(f_n\) is call the Fibonacci sequence.

24

Prove that

  1. For every positive integer \(n\text{,}\) \(f_{3n}\) is even and \(f_{3n+1},f_{3n+2}\) are both odd.
  2. For every positive integer \(n\text{,}\) \(\gcd(f_n,f_{n+1}) = 1\) and \(\gcd(f_n,f_{n+2}) = 1\text{.}\)
  3. For every positive integer \(n\text{,}\) \begin{equation*} \sum_{j=1}^n f_j = f_{n+2} - 1. \end{equation*}
Solution

For each of these, we will use strong induction.

  1. Notice that \(f_1 = 1 = f_2\text{,}\) so these are both odd. Now suppose that \(m \ge 3\) and for any \(k \lt m\text{,}\) \(f_k\) is even if and only if \(k\) is divisible by \(3\text{.}\) Now we apply the division algorithm to divide \(m\) by \(3\) to get \(m = 3q + r\) for some \(q,r \in \mathbb{Z}\) with \(r \in \{0,1,2\}\text{.}\) Case 1: \(r = 0\text{.}\) Then \(f_m = f_{m-1} + f_{m-2} = f_{3(q-1)+2} + f_{3(q-1)+1}\) and by the strong inductive hypothesis, these last two terms are odd since the subscripts are not divisible by \(3\text{,}\) and therefore their sum is even. Case 2: \(r = 1\text{.}\) Then \(f_m = f_{m-1} + f_{m-2} = f_{3q} + f_{3(q-1)+2}\) and by the strong inductive hypothesis, \(f_{3q}\) is even and \(f_{3(q-1)+2}\) is odd, therefore their sum is odd. Case 2: \(r = 2\text{.}\) Then \(f_n = f_{m-1} + f_{m-2} = f_{3q+1} + f_{3q}\) and by the strong inductive hypothesis, \(f_{3q}\) is even and \(f_{3q+1}\) is odd, therefore their sum is odd. Thus \(f_m\) is even if and only if \(m\) is divisible by \(3\text{.}\) By strong induction, the proof is complete.
  2. Notice that \(f_1 = f_2 = 1\) and \(f_3 = 2\text{,}\) and so \(\gcd(f_1,f_2) = \gcd(1,1) = 1\) and similarly, \(\gcd(f_1,f_3) = \gcd(1,2) = 1\text{.}\) Now, let \(m \ge 2\) and suppose that for any \(k \lt m\text{,}\) we have \(\gcd(f_k,f_{k+1}) = 1 = \gcd(f_k,f_{k+2})\text{.}\) Then we find that \(f_{m+1} = f_m + f_{m-1}\text{.}\) Let \(c\) be any common divisor of \(f_m,f_{m+1}\text{.}\) Then it is also a divisor of \(f_{m-1} = f_{m+1} - f_m\text{,}\) so it is a common divisor of \(f_{m-1},f_m\text{.}\) Therefore, by the strong inductive hypothesis, \(c = 1\text{.}\) Hence \(\gcd(f_m,f_{m+1}) = 1\text{.}\) Now let \(c\) be any common divisor of \(f_m,f_{m+2}\text{.}\) Since \(f_{m+2} = f_{m+1} + f_m\text{,}\) we see that \(c\) is also a divisor of \(f_{m+1} = f_{m+2} - f_m\text{.}\) Therefore it is a common divisor of \(f_m,f_{m+1}\text{.}\) By the previous paragraph, this implies that \(c = 1\text{.}\) Hence \(\gcd(f_m,f_{m+2}) = 1\text{.}\) By strong induction, we have established the result.
  3. Notice that \(\sum_{j=1}^1 f_j = f_1 = 1 = 2 - 1 = f_{1+2}-1\text{.}\) For the inductive step, assume that \(m \gt 1\) and for any \(k \lt m\text{,}\) \(\sum_{j=1}^k f_j = f_{k+2}-1\text{.}\) Then \begin{align*} f_{m+2} - 1 &= f_{m+1} + f_m - 1\\ &= \sum_{j=1}^{m-1} f_j + f_m - 1\\ &= \sum_{j=1}^m f_j - 1. \end{align*} By strong induction we have proven the result.
25

Let \(\alpha,\beta\) be the (positive, negative) solutions to the quadratic equation \(x^2 = x+1\text{.}\) That is, \(\alpha = \frac{1+\sqrt{5}}{2}\) and \(\beta = \frac{1-\sqrt{5}}{2}\text{.}\) Prove that for every \(n \in \mathbb{N}\text{,}\)

\begin{equation*} f_n = \frac{\alpha^n - \beta^n}{\alpha - \beta}. \end{equation*}
Solution

We will prove this by strong induction. Notice that \(\frac{\alpha^1 - \beta^1}{\alpha - \beta} = 1 = f_1\) and also

\begin{equation*} \frac{\alpha^2 - \beta^2}{\alpha - \beta} = \alpha + \beta = 1 = f_2. \end{equation*}

Now let \(m \ge 3\) and assume that for all \(k \lt m\text{,}\) we have

\begin{equation*} f_k = \frac{\alpha^k - \beta^k}{\alpha - \beta}. \end{equation*}

Then

\begin{align*} f_m &= f_{m-1} + f_{m-2}\\ &= \frac{\alpha^{m-1} - \beta^{m-1}}{\alpha - \beta} + \frac{\alpha^{m-2} - \beta^{m-2}}{\alpha - \beta}\\ &= \frac{\alpha^{m-2}(\alpha + 1) - \beta^{m-2}(\beta + 1)}{\alpha - \beta}\\ &= \frac{\alpha^{m-2}(\alpha^2) - \beta^{m-2}(\beta^2)}{\alpha - \beta}\\ &= \frac{\alpha^m - \beta^m}{\alpha - \beta}. \end{align*}
26

Let \(a_1 = 2, a_2 = 4\) and \(a_{n+2} = 5a_{n+1} - 6a_n\) for all \(n \ge 1\text{.}\) Prove that \(a_n = 2^n\) for every \(n \in \mathbb{N}\text{.}\)

Solution

Notice that \(a_1 = 2 = 2^1\) and \(a_2 = 4 = 2^2\text{.}\) Now let \(m \ge 3\) and suppose that for all \(k \lt m\text{,}\) \(a_k = 2^k\text{.}\) Then by the definition of \(a_m\) and the strong inductive hypothesis, we find

\begin{align*} a_m &= 5a_{m-1} - 6a_{m-2}\\ &= 5\cdot 2^{m-1} - 6 \cdot 2^{m-2}\\ &= 5\cdot 2^{m-1} - 3 \cdot 2^{m-1}\\ &= (5-3)\cdot 2^{m-1}\\ &= 2^m \end{align*}
27

For every natural number \(n\text{,}\) \(5\) divides \(8^n - 3^n\text{.}\)

Solution

Notice that \(5\) obviously divides \(5 = 8-3 = 8^1-3^1\text{.}\) Now suppose \(m \in \mathbb{N}\) and assume \(5\) divides \(8^m-3^m\) so that \(8^m-3^m =5k\) for some integer \(k\text{.}\) Then

\begin{align*} 8^{m+1}-3^{m+1} &= 8 \cdot 8^m - 3 \cdot 3^m\\ &= 8 \cdot 8^m - 3 \cdot 3^m - 5 \cdot 3^m + 5\cdot 3^m \\ &= 8 \cdot 8^m - (3+5) \cdot 3^m + 5\cdot 3^m \\ &= 8 (8^m - 3^m) + 5\cdot 3^m \\ &= 8\cdot 5k + 5\cdot 3^m \\ &= 5(8k + 3^m). \end{align*}
28

Every natural number \(n \gt 1\) has a prime divisor.

Solution

Suppose to the contrary that not every natural number greater than \(1\) has a prime divisor. Then by the Well-Ordering Principle, there is a smallest such natural number, which we will call \(n\text{.}\) Note that \(n\) cannot be prime, for otherwise it would be its own prime divisor. Therefore, since \(n \gt 1\text{,}\) it must be composite, and thus \(n = mk\) for \(1 \lt m,k \lt n\text{.}\) By the minimality of \(n\text{,}\) \(k\) must have a prime divisor, which we call \(p\text{.}\) Thus \(k=xp\) for some integer \(x\text{.}\) Finally, \(n=mk=mxp\) and hence \(p\) is a prime divisor of \(n\text{,}\) which is a contradiction. Therefore, every natural number greater than \(1\) has a prime divisor.

29

Give examples of relations (on any set or sets of your choosing) which satisfy each of the following combinations of properties.

  1. neither reflexive nor irreflexive
  2. reflexive and symmetric, but not transitive
  3. reflexive and antisymmetric, but not transitive
  4. reflexive and transitive, but not symmetric
Solution

  1. \(A = \{0,1\}\text{,}\) \(R = \{(0,0)\}\text{.}\) \(R\) is not reflexive because \(1\,\not R\,1\text{,}\) and it is not irreflexive because \(0\,R\,0\text{.}\)
  2. \(A = \{0,1,2,\}\text{,}\) \(R = \{(0,0),(1,1),(2,2),(0,1),(1,0),(1,2),(2,1)\}\text{.}\) This is clearly reflexive and symmetric, but it is not transitive because \(0\,R\,1\) and \(1\,R\,2\) but \(0\,\not R\,2\text{.}\)
  3. \(A = \{0,1,2,\}\text{,}\) \(R = \{(0,0),(1,1),(2,2),(0,1),(1,2)\}\text{.}\) This is clearly reflexive and antisymmetric, but it is not transitive because \(0\,R\,1\) and \(1\,R\,2\) but \(0\,\not R\,2\text{.}\)
  4. \(A = \{0,1,2,\}\text{,}\) \(R = \{(0,0),(1,1),(2,2),(0,1),(1,2),(0,2)\}\text{.}\) This is reflexive and transitive, but it is not symmetric because \(0\,R\,1\) but \(1\,\not R\,0\text{.}\) As an alternative example you could take \(\le\) on \(\mathbb{R}\text{.}\)
Slopes and lines
30

Let \(r \in \mathbb{R}\) be a fixed real number. Define a relation \(\sim_r\) on \(\mathbb{R}^2\) in the following way. For any distinct points \((x,y), (w,z) \in \mathbb{R}^2\text{,}\) set \((x,y) \sim_r (w,z)\) if and only if the slope of the line through these two points is \(r\text{.}\) Moreover, set \((x,y) \sim_r (x,y)\) always. Prove that \(\sim_r\) is an equivalence relation (note: reflexivitiy is very easy).

Solution

This relation \(\sim_r\) is reflexive by definition.

For symmetry, let \((x,y), (w,z) \in \mathbb{R}^2\) be any distinct points with \((x,y) \sim_r (w,z)\text{.}\) Thus the slope of the line through these points is \(r\text{,}\) i.e.

\begin{equation*} \frac{z-y}{w-x} = r. \end{equation*}

Now let us compute the slope of the line through the points in the other order:

\begin{equation*} \frac{y-z}{x-w} = \frac{-(z-y)}{-(w-x)} = \frac{z-y}{w-x} = r. \end{equation*}

For transitivity, suppose that \((x,y) \sim_r (w,z) \sim_r (a,b)\text{.}\) Then let's look at the slope of the line through \((x,y),(a,b)\text{:}\)

\begin{align*} \frac{b-y}{a-x} = \frac{(b-z)+(z-y)}{a-x} &= \frac{b-z}{a-x} + \frac{z-y}{a-x}\\ &= \frac{b-z}{a-x} \cdot \frac{a-w}{a-w} + \frac{z-y}{a-x} \cdot \frac{w-x}{w-x}\\ &= r \frac{a-w}{a-x} + r \frac{w-x}{a-x} = r \frac{a-w+w-x}{a-x} = r. \end{align*}

Thus \((x,y) \sim_r (a,b)\text{.}\)

31

When \(r = 5\text{,}\) describe the equivalence class \([(3,10)]\text{.}\)

Solution

The equivalence class \([(3,10)]\) is the set of points such that the line through that point and \([(3,10)]\) has slope \(5\text{.}\) However, each of these lines passes through \((3,10)\) and have the slope \(5\text{,}\) but there is only one such line. In particular, the equivalence class is this line. In other words,

\begin{equation*} [(3,10)] = \{ (x,y) \in \mathbb{R}^2 \mid y = 5(x-3)+10 \}. \end{equation*}
32

Give a geometric description of the partition of \(\mathbb{R}^2\) induced by the equivalence relation \(\sim_r\text{.}\)

Solution

The elements of the partition induced by \(\sim_r\) are just all the lines of slope \(\sim_r\) in the plane.

33

This is problem 3 on page 162 of your textbook.

Describe the partition for each of the following equivalence relations.

  1. For \(x,y \in \mathbb{R}\text{,}\) \(x \sim y\) iff \(x-y \in \mathbb{Z}\text{.}\)
  2. For \(x,y \in \mathbb{R}\text{,}\) \(x \sim y\) iff \(\sin x = \sin y\text{.}\)
  3. For \(x,y \in \mathbb{R}\text{,}\) \(x \sim y\) iff \(x^2 = y^2\text{.}\)
  4. For \((x,y), (u,v) \in \mathbb{R}^2\text{,}\) \((x,y) \sim (u,v)\) iff either \(xy = uv = 0\) or \(xyuv \gt 0\text{.}\)
Solution

  1. The equivalence classes consist of sets of points whose two nearest neighbors are exactly distance 1 apart. One of these equivalence class is just \(\mathbb{Z}\) itself, and the others are just shifted copies of \(\mathbb{Z}\text{.}\)
  2. Notice that \(\sin x = \sin y\) if and only if either \(y = x + 2\pi k\) for some integer \(k\) or if \(y = (\pi - x) +2\pi k\) for some integer \(k\text{.}\) So the equivalence classes are the union of two sets of points each having an equal spacing of \(2\pi\text{.}\) Alternatively, you can view the equivalence classes as the set of points of intersection of a horizontal line (between \(-1\) and \(1\) on the vertical axis) with the graph of \(\sin(x)\text{.}\)
  3. The equivalence classes from this equivalence relation are just the union of a point with its negative, e.g. \(\{2,-2\}\text{.}\) These are all two point sets except the line \(\{0\}\) which is a singleton. As in the previous example, you can also think of this as the set of points of intersection of a horizontal line with the graph of the parabola \(x^2\text{.}\)
  4. Given two points \((x,y), (u,v)\text{,}\) \(xy = 0\) if and only if either \(x=0\) or \(y =0\text{,}\) that is, if \((x,y)\) lies on one of the axes. So, two points are similar if either \(xy = uv = 0\) (i.e. they both lie on the axes) or \(xyuv \gt 0\) (which can only happen if both points are not on the axes). Thus the axes are one of the partitions. Now, how can \(xyuv \gt 0\text{?}\) Consider the different quadrants: if \((x,y)\) is in: quadrant 1, then \(xy \gt 0\text{;}\) quadrant 2, then \(xy \lt 0\text{;}\) quadrant 3, then \(xy \gt 0\text{;}\) quadrant 4, then \(xy \lt 0\text{.}\) So, the only way for \(xyuv \gt 0\) is if both \((x,y),(u,v)\) are in quadrants 1,3 or both are in quadrants 2,4. Thus the partition has 3 sets, the axes, the union of quadrants 1 and 3, and the union of quadrants 2 and 4.
The integers, \(\mathbb{Z}\)
34

Consider the following relation on \(\mathbb{N}^2\text{.}\) For \((x,y), (u,v) \in \mathbb{N}^2\text{,}\) \((x,y) \sim (u,v)\) iff \(x+v = y+u\text{.}\) Prove that \(\sim\) is an equivalence relation.

Solution

Reflexivity: since \(x+y = y+x\) for any \((x,y) \in \mathbb{N}^2\text{,}\) then \((x,y) \sim (x,y)\text{.}\)

Symmetry: if \((x,y) \sim (u,v)\text{,}\) then \(x+v = y+u\) so \(u+z = w+v\text{,}\) thus \((u,v) \sim (x,y)\text{.}\)

Transitivitiy: if \((x,y) \sim (u,v) \sim (w,z)\) then \(x+v+z = y+u+z = y+w+v\text{.}\) By cancelling the \(v\) we obtain \(x+z = w+y\) and hence \((x,y) \sim (w,z)\text{.}\)

35

Describe the equivalence class \([(1,1)]\text{.}\)

Solution

Note that \((n,m) \in [(1,1)]\) if and only if \(n+1 = m+1\) if and only if \(n = m\text{.}\) Thus \([(1,1)] = \{(n,n) \mid n \in \mathbb{N}\}\text{.}\)

36

Show that every equivalence class has one of the following forms:

  • \([(n+1,1)]\) for some \(n \in \mathbb{N}\text{.}\)
  • \([(1,1)]\text{.}\)
  • \([(1,n+1)]\) for some \(n \in \mathbb{N}\text{.}\)

What common set do you think it would be natural to identify with the set of equivalence classes \(\mathbb{N}^2 / \sim\text{?}\)

Solution

Consider the equivalence class \([(n,m)]\text{.}\) If \(n=m\) then we have already shown that this is \([(1,1)]\) in the previous problem.

If \(n \lt m\text{,}\) then there is some \(k \in \mathbb{N}\) for which \(n+k = m\text{.}\) Thus \(n+(k+1) = m+1\text{,}\) and hence \((n,m) \sim (1,k+1)\) so their equivalence class are the same.

If \(n \gt m\text{,}\) then just reverse their roles in the previuos paragraph.

The rational numbers, \(\mathbb{Q}\)
37

Consider the relation \(\sim\) on the set \(\mathbb{Z} \times (\mathbb{Z} \setminus \{0\})\) defined by \((p,q) \sim (a,b)\) if and only if \(pb = aq\text{.}\) Prove that \(\sim\) is an equivalence relation.

Solution

Reflexivity: since \(pq = pq\text{,}\) we know \((p,q) \sim (p,q)\text{.}\)

Symmetry: suppose \((p,q) \sim (a,b)\text{,}\) then \(pb = aq\text{,}\) and so obviously \(aq = pb\text{,}\) hence \((a,b) \sim (p,q)\text{.}\)

Transitivity: suppose \((p,q) \sim (a,b) \sim (r,s)\text{,}\) then \(pb = aq\) and \(as = rb\text{.}\) Then \(pbs = aqs = qrb\) and since \(b \not= 0\) we may cancel it to obtain \(ps = rq\text{.}\) Hence \((p,q) \sim (r,s)\text{.}\)

38

Prove that in every equivalence class \([(p,q)]\) with \(p \not= 0\) there is some element \((a,b)\) in this equivalence class for which \(\gcd(a,b) = 1\text{.}\) Moreover, prove that if we add the constraint that \(b \gt 0\) then this element is actually unique.

Solution

Recall from a previous homework exercise that if \(p,q \not= 0\) and \(d = \gcd(p,q)\text{,}\) then \(\gcd(\frac{p}{d}, \frac{q}{d}) = 1\text{.}\) (Note: here what we mean by \(\frac{p}{d}\) is the quotient from the division algorithm, and similarly for \(\frac{q}{d}\text{.}\)) Set \(a = \frac{p}{d}\) and \(b = \frac{q}{d}\text{.}\) Since \(p,q \not= 0\) so also \(a,b \not= 0\text{.}\) Finally, notice that \(pbd = ab = aqd\) and by cancellation, we may drop the \(d\) to obtain \(pb = aq\text{.}\)

Now, suppose that there are two of these elements in a given equivalence class with the second coordinate positive. Call them \((a,b) \sim (p,q)\) and \(\gcd(a,b) = \gcd(p,q) = 1\) and \(b,q \gt 0\text{.}\) Then \(pb = aq\text{.}\) Mutliplying by \(a\) we obtain \(pab = a^2 q\text{.}\) Since \(\gcd(a,b) = 1\text{,}\) and \(ab\) divides \(a^2 q\text{,}\) by a result proved after the first exam for relatively prime numbers, we can conclude that either \(ab\) divides \(a^2\) (in which case \(a = b = 1\) and hence also \(p = q = 1\)) or \(ab\) divides \(q\text{.}\) If the former, then we are done, so assume \(ab\) divides \(q\text{.}\) Then there is some integer \(k\) so that \(q = abk\text{,}\) and hence \(pb = aq = a^2bk\text{.}\) But then \(ab\) divides \(pb\) and so by that same earlier result, \(ab\) divides \(p\) or \(b\text{.}\) If it divides \(b\text{,}\) then everything collapses and we are done. If it divides \(p\text{,}\) then \(p,q\) would have a common factor, \(ab\) violating \(\gcd(p,q) = 1\) unless \(a = b = 1\text{.}\)

39

Consider the following relation \(R,S,T,U\) on \(A = \{0,1,2\}\text{.}\)

\begin{align*} R &= \{ (0,0), (0,1), (1,1), (1,2), (2,2), (0,2), (1,0) \}.\\ S &= \{ (0,0), (0,1), (1,1), (1,2), (2,2), (0,2) \}.\\ T &= \{ (0,0), (0,1), (1,1) \}.\\ U &= \{ (0,0), (0,1), (1,1), (1,2), (2,2) \}. \end{align*}

Are any of \(R,S,T,U\) a partial order on \(A\text{?}\) Provide justification.

Solution

\(R\) is not antisymmetric since \((0,1),(1,0) \in R\text{.}\)

\(S\) is a partial order because it is reflexive (contains \((0,0),(1,1),(2,2)\)), antisymmetric (contains \((0,1),(1,2),(0,2)\) but not any of \((1,0),(2,1),(2,0)\)), and transitive (it contains \((0,1),(1,2)\) and also \((0,2)\)). Note that \(S\) is just the partial order \(\le\) on \(A\text{.}\)

\(T\) is not reflexive since \((2,2) \notin T\text{.}\)

\(U\) is not transitive since \((0,1),(1,2) \in U\) but \((0,2) \notin U\text{.}\)

40

Define a relation \(\prec\) on \(\mathbb{R}^2\) by \((a,b) \prec (x,y)\) if and only if \(a \le x\) and \(b \le y\text{.}\) Prove that \(\prec\) is a partial order on \(\mathbb{R}^2\text{.}\) Is it a total order?

Solution

We first prove that \(\prec\) is reflexive. Note that for any \((x,y) \in \mathbb{R}^2\text{,}\) we have \(x \le x\) and \(y \le y\text{,}\) and therefore \((x,y) \prec (x,y)\text{.}\)

To see that \(\prec\) is antisymmetric, note that for any \((x,y),(a,b) \in \mathbb{R}^2\text{,}\) if \((x,y) \prec (a,b)\) and \((a,b) \prec (x,y)\text{,}\) then \(x \le a\) amd \(y \le b\text{,}\) and also \(a \le x\) and \(b \le y\text{.}\) Therefore, by the antisymmetry of \(\le\text{,}\) we find \(x = a\) and \(y = b\text{,}\) therefore \((x,y) = (a,b)\text{.}\)

To see that \(\prec\) is transitive, note that for any \((x,y), (a,b), (w,z) \in \mathbb{R}^2\text{,}\) if \((x,y) \prec (a,b)\) and \((a,b) \prec (w,z)\text{,}\) then \(x \le a\) and \(y \le b\text{,}\) and also \(a \le w\) and \(b \le z\text{.}\) Therefore, by the transitivity of \(\le\text{,}\) we find \(x \le w\) and \(y \le z\text{.}\) Hence \((x,y) \prec (w,z)\text{.}\)

We have shown that \(\prec\) is a partial order on \(\mathbb{R}^2\) because it is reflexive, antisymmetric and transitive, but it is not a total order because there are incomparable elements. In particular, \((1,2) \not\prec (0,3)\) since \(1 \not\le 0\text{,}\) but also \((0,3) \not\prec (1,2)\) since \(3 \not\le 2\text{.}\)

41

Define the relation \(R\) on \(\mathbb{C}\) by \((a+bi) R (c+di)\) if and only if \(a^2 + b^2 \le c^2 + d^2\text{.}\) Is \(R\) a partial order on \(\mathbb{C}\text{?}\) Justify your answer.

Solution

The relation \(R\) is not a partial order on \(\mathbb{C}\) because it is not antisymmetric. In particular, \(1\ R\ i\) and \(i\ R\ 1\) since \(1^2 + 0^2 = 0^2 + 1^2\text{,}\) but \(1 \not= i\text{.}\) This completes the solution to this problem.

Note however that this relation is reflexive and transitive. A relation like this is called a pre-order. If \(S\) is any pre-order (reflexive, transitive) on a set \(A\text{,}\) then we can create an equivalence relation \(\sim\) on \(A\) by \(x \sim y\) if and only if \(x\ S\ y\) and \(y\ S\ x\text{.}\) This is easily seen to be symmetric, and the reflexivity and transitivity follow from those same properties of \(S\text{.}\) Then there is a natural partial order \(S/\sim\) on the set of equivalence classes \(A/\sim\) defined by \([x] S/\sim [y]\) if and only if \(x\ S\ y\text{.}\)

If we were to apply the procedure in the last paragraph to the relation \(R\) given in this problem, then the equivalence relation on \(\mathbb{C}\) we would obtain is the one in which \(x \sim y\) if and only if \(x,y\) lie on the same circle centered at \(0\text{.}\) So the equivalence classes are circles centered at \(0\text{.}\) Then the partial order \(R/\sim\) on the set of these circles is precisely the one that says one circle is less than or equal to another circle if and only if its radius is less than or equal to the radius of the other.